POLI REV - IV. Executive Department Case Digests

You might also like

Download as doc, pdf, or txt
Download as doc, pdf, or txt
You are on page 1of 19

POLITICAL LAW REVIEW: IV.

Executive Department Case Digests | Wednesday Class @ 4:30-9:30PM

IV. Executive Department While petitioner concedes that the Supreme Court is
A.2. Presidential Privilege "authorized to promulgate its rules for the purpose," he chafes at
the creation of a purportedly "separate tribunal" complemented
1. Pormento vs Estrada by a budget allocation, a seal, a set of personnel and confidential
G.R. No. 191988, August 31, 2010 employees, to effect the constitutional mandate.
ATTY. EVILLO C. PORMENTO, versus JOSEPH ERAP
EJERCITO ESTRADA and COMMISSION ON ELECTIONS, ISSUE: Is the PET unconstitutional?
FACTS:
The petition asks whether private respondent Joseph Ejercito RULING: NO. The conferment of additional jurisdiction to the
Estrada is covered by the ban on the President from any Supreme Court, with the duty characterized as an "awesome"
reelection. Private respondent was elected President of the task, includes the means necessary to carry it into effect under
Republic of the Philippines in the general elections held on May the doctrine of necessary implication.
11, 1998. He sought the presidency again in the general elections A plain reading of Article VII, Section 4, paragraph 7,
held on May 10, 2010. Petitioner Atty. Evillo C. Pormento opposed readily reveals a grant of authority to the Supreme Court sitting
private respondents candidacy and filed a petition for en banc. In the same vein, although the method by which the
disqualification. Supreme Court exercises this authority is not specified in the
ISSUE: What is the proper interpretation of the following provision provision, the grant of power does not contain any limitation on
of Section 4, Article VII of the Constitution: [t]he President shall not the Supreme Courts exercise thereof. The Supreme Court's
be eligible for any reelection? method of deciding presidential and vice-presidential election
HELD: contests, through the PET, is actually a derivative of the exercise
Private respondent was not elected President the second time he of the prerogative conferred by the aforequoted constitutional
ran. Since the issue on the proper interpretation of the phrase any provision. Thus, the subsequent directive in the provision for the
reelection will be premised on a persons second (whether Supreme Court to "promulgate its rules for the purpose."
immediate or not) election as President, there is no case or The conferment of full authority to the Supreme Court,
controversy to be resolved in this case. No live conflict of legal as a PET, is equivalent to the full authority conferred upon the
rights exists.[6] There is in this case no definite, concrete, real or electoral tribunals of the Senate and the House of
substantial controversy that touches on the legal relations of Representatives, i.e., the Senate Electoral Tribunal (SET) and
parties having adverse legal interests.[7] No specific relief may the House of Representatives Electoral Tribunal (HRET), which
conclusively be decreed upon by this Court in this case that will we have affirmed on numerous occasions.
benefit any of the parties herein.[8] As such, one of the essential Republic Act No. 1793 has not created a new or
requisites for the exercise of the power of judicial review, the separate court. It has merely conferred upon the Supreme Court
existence of an actual case or controversy, is sorely lacking in this the functions of a Presidential Electoral Tribunal. The result of
case. the enactment may be likened to the fact that courts of first
As a rule, this Court may only adjudicate actual, ongoing instance perform the functions of such ordinary courts of first
controversies.[9] The Court is not empowered to decide moot instance, those of court of land registration, those of probate
questions or abstract propositions, or to declare principles or rules courts, and those of courts of juvenile and domestic relations. It
of law which cannot affect the result as to the thing in issue in the is, also, comparable to the situation obtaining when the municipal
case before it.[10] In other words, when a case is moot, it becomes court of a provincial capital exercises its authority, pursuant to
non-justiciable.[11] law, over a limited number of cases which were previously within
An action is considered moot when it no longer presents a the exclusive jurisdiction of courts of first instance.
justiciable controversy because the issues involved have become By the same token, the PET is not a separate and
academic or dead or when the matter in dispute has already been distinct entity from the Supreme Court, albeit it has functions
resolved and hence, one is not entitled to judicial intervention peculiar only to the Tribunal. It is obvious that the PET was
unless the issue is likely to be raised again between the parties. constituted in implementation of Section 4, Article VII of the
There is nothing for the court to resolve as the determination Constitution, and it faithfully complies not unlawfully defies the
thereof has been overtaken by subsequent events.[12] constitutional directive. The adoption of a separate seal, as well
Assuming an actual case or controversy existed prior to the as the change in the nomenclature of the Chief Justice and the
proclamation of a President who has been duly elected in the May Associate Justices into Chairman and Members of the Tribunal,
10, 2010 elections, the same is no longer true today. Following the respectively, was designed simply to highlight the singularity and
results of that elections, private respondent was not elected exclusivity of the Tribunals functions as a special electoral court.
President for the second time. Thus, any discussion of his We have previously declared that the PET is not simply
reelection will simply be hypothetical and speculative. It will serve an agency to which Members of the Court were designated.
no useful or practical purpose. Once again, the PET, as intended by the framers of the
Constitution, is to be an institution independent, but not separate,
2. Macalintal vs PET from the judicial department, i.e., the Supreme Court.
ATTY. ROMULO B. MACALINTAL vs. PRESIDENTIAL
ELECTORAL TRIBUNAL (PET)
G.R. No. 191618               November 23, 2010
NACHURA, J.

FACTS: Atty. Romulo B. Macalintal filed a petition before the SC,


challenging the constitutionality of Presidential Electoral Tribunal
(PET) as an illegal and unauthorized progeny of Section 4, Article
VII of the Constitution, which provides:
The Supreme Court, sitting en banc, shall be the sole
judge of all contests relating to the election, returns, and
qualifications of the President or Vice-President, and may
promulgate its rules for the purpose.

P a g e 1 | 19
POLITICAL LAW REVIEW: IV. Executive Department Case Digests | Wednesday Class @ 4:30-9:30PM

ATTY. ROMULO B. MACALINTAL vs. PRESIDENTIAL 5. De Castro vs. Judicial Bar Council and Pres. Macapagal-
ELECTORAL TRIBUNAL (PET) Arroyo
G.R. No. 191618               June 7,2011
NACHURA, J. Facts:
FACTS: A Motion for Reconsideration filed by petitioner Atty.
Romulo B. Macalintal of our Decision[1] in G.R. No. 191618 dated The forthcoming compulsory retirement of Chief Justice Puno
November 23, 2010, dismissing his petition and declaring the occurred 7 days after the Presidential election. Under Sec. 4(1),
establishment of respondent Presidential Electoral Tribunal (PET) Art. 8 of the Constitution, provides that vacancy shall be filed
as constitutional. within 90 days from the occurrence thereof from the list of at
Petitioner reiterates his arguments on the alleged least 3 nominees prepared by the Judicial and Bar Council for
unconstitutional creation of the PET such as a) Section 4, Article every vacancy.
VII of the Constitution does not provide for the creation of the PET
and b) The PET violates Section 12, Article VIII of the Constitution. Cong. Matias Defensor, an ex officio member of JBC, addressed
On the other hand, in its Comment to the Motion for a letter to the JBC, requesting that the process for nominations to
Reconsideration, the Office of the Solicitor General maintains that t the office of the Chief Justice be commenced immediately.
he constitution of the PET is on firm footing on the basis of the
grant of authority to the Supreme Court to be the sole judge of all JBC passed a resolution agreeing to start the filling of the
election contests for the President or Vice-President under position. Application or recommendation and publication opened.
paragraph 7, Section 4, Article VII of the 1987 Constitution.
Conformably, JBC automatically considered the position of the 5
ISSUE: IS the PET unconstitutional? most senior associate justices of the court. Others either applied
or were nominated.
RULING: NO. The discussions of the Constitutional Commission
unmistakably indicate the exercise of the SC’s power to judge Although it has already begun the process of filling of the
presidential and vice-presidential election contests, as well as the position, the JBC is not yet decided on when to submit to the
rule-making power adjunct thereto, is plenary; it is not as restrictive President its list of nominees. The actions of the JBC have
as petitioner would interpret it. In fact, former Chief Justice Hilario sparked vigorous debate on whether the incumbent President
G. Davide, Jr., who proposed the insertion of the phrase, intended can appoint the next Chief Justice or not.
the Supreme Court to exercise exclusive authority to promulgate
its rules of procedure for that purpose. To this, Justice Regalado The OSG submitted its comment stating that the incumbent
forthwith assented and then emphasized that the sole power ought President can appoint the successor of the Chief Justice Puno
to be without intervention by the legislative department. Evidently, upon his retirement because the prohibition under Sec. 15, Art. 7
even the legislature cannot limit the judicial power to resolve of the Constitution does not apply to appointments to the SC. It
presidential and vice-presidential election contests and our rule- argues that any vacancy in the SC must be filed within 90 days
making power connected thereto. from its occurrence, pursuant to Sec. 4(1), Art. 8 of the
To foreclose all arguments of petitioner, we reiterate that Constitution; that in their deliberations on the mandatory period
the establishment of the PET simply constitutionalized what was for the appointment of the SC Justices, the framers neither
statutory before the 1987 Constitution. The experiential context of mentioned nor referred to the ban against midnight
the PET in our country cannot be denied appointments, or its effects on such period, or vice versa; that
Furthermore, judicial power granted to the Supreme Court had the framers intended the prohibition to apply to SC
by the same Constitution is plenary. And under the doctrine of appointments, they could have easily expressly stated so in the
necessary implication, the additional jurisdiction bestowed by the Constitution, which explains why the prohibition found in Art. 7
last paragraph of Section 4, Article VII of the Constitution to decide (executive department) was not written in Art 8 (judicial
presidential and vice-presidential elections contests includes the department); and that the framers also incorporated in Art. 8
means necessary to carry it into effect.  ample restrictions or limitations on the President power to
The traditional grant of judicial power is found in Section appoint members of the SC to ensure its independence from
1, Article VIII of the Constitution which provides that the power political vicissitudes and its insulation from political pressures,
“shall be vested in one Supreme Court and in such lower courts as such as stringent qualifications for the positions, the
may be established by law.” The set up embodied in the establishment of the JBC, the specified period within which the
Constitution and statutes characterize the resolution of electoral President shall appoint a SC Justice.
contests as essentially an exercise of judicial power. When the
Supreme Court, as PET, resolves a presidential or vice- The Court admitted the comments.
presidential election contest, it performs what is essentially a
judicial power. The main question presented in all the filings herein because it
The COMELEC, HRET and SET are not, strictly and involves 2 seemingly conflicting provisions of the Constitution
literally speaking, courts of law. Although not courts of law, they imperatively demands the attention and resolution of this Court,
are, nonetheless, empowered to resolve election contests which the only authority that can resolve the question definitively and
involve, in essence, an exercise of judicial power, because of the finally. Thus, we resolve.
explicit constitutional empowerment found in Section 2(2), Article
IX-C (for the COMELEC) and Section 17, Article VI (for the Senate Issue:
and House Electoral Tribunals) of the Constitution.
Whether or not the incumbent President can appoint the
successor of Chief Justice Puno upon his retirement?
C. Rules of Succession
Ruling: YES.
3. Funa vs Ermita
4. Funa vs Agra Arturo Prohibition under Section 15, Art 7 does not apply to
appointments to fill vacancy in the Supreme Court or to other
appointments to the Judiciary.
P a g e 2 | 19
POLITICAL LAW REVIEW: IV. Executive Department Case Digests | Wednesday Class @ 4:30-9:30PM

or Acting Presidents term does not refer to the Members of the


2 constitutional provisions are seemingly in conflict. Supreme Court.

The first, Section 15, Article VII (Executive Department), provides: Moreover, the usage in Section 4(1), Article VIII of the word shall
an imperative, operating to impose a duty that may be enforced
Section 15. Two months immediately before the next presidential should not be disregarded. Thereby, Sections 4(1) imposes on
elections and up to the end of his term, a President or Acting the President the imperative duty to make an appointment of a
President shall not make appointments, except temporary Member of the Supreme Court within 90 days from the
appointments to executive positions when continued vacancies occurrence of the vacancy. The failure by the President to do so
therein will prejudice public service or endanger public safety. will be a clear disobedience to the Constitution.

The other, Section 4 (1), Article VIII (Judicial Department), states: The 90-day limitation fixed in Section 4(1), Article VIII for the
President to fill the vacancy in the Supreme Court was
Section 4. (1). The Supreme Court shall be composed of a Chief undoubtedly a special provision to establish a definite mandate
Justice and fourteen Associate Justices. It may sit en banc or in its for the President as the appointing power, and cannot be
discretion, in division of three, five, or seven Members. Any defeated by mere judicial interpretation.
vacancy shall be filled within ninety days from the occurrence
thereof. Consequently, prohibiting the incumbent President from
appointing a Chief Justice on the premise that Section 15, Article
In the consolidated petitions, submit that the incumbent President VII extends to appointments in the Judiciary cannot be sustained.
can appoint the successor of Chief Justice Puno upon his
retirement on the ground that the prohibition against presidential Second. Section 15, Article VII does not apply as well to all other
appointments under Section 15, Article VII does not extend to appointments in the Judiciary.
appointments in the Judiciary.
There is no question that one of the reasons underlying the
The Court agrees with the submission. adoption of Section 15 as part of Article VII was to eliminate
midnight appointments from being made by an outgoing Chief
First. The records of the deliberations of the Constitutional Executive.
Commission reveal that the framers devoted time to meticulously
drafting, styling, and arranging the Constitution. Such Given the background and rationale for the prohibition in Section
meticulousness indicates that the organization and arrangement of 15, Article VII, we have no doubt that the Constitutional
the provisions of the Constitution were not arbitrarily or whimsically Commission confined the prohibition to appointments made in
done by the framers, but purposely made to reflect their intention the Executive Department. The framers did not need to extend
and manifest their vision of what the Constitution should contain. the prohibition to appointments in the Judiciary, because their
establishment of the JBC and their subjecting the nomination and
The Constitution consists of 18 Articles, three of which embody the screening of candidates for judicial positions to the unhurried and
allocation of the awesome powers of government among the three deliberate prior process of the JBC ensured that there would no
great departments, the Legislative (Article VI), the Executive longer be midnight appointments to the Judiciary.
(Article VII), and the Judicial Departments (Article VIII). The
arrangement was a true recognition of the principle of separation Also, the intervention of the JBC eliminates the danger that
of powers that underlies the political structure. appointments to the Judiciary can be made for the purpose of
buying votes in a coming presidential election, or of satisfying
As can be seen, Article VII is devoted to the Executive partisan considerations. The experience from the time of the
Department, and, among others, it lists the powers vested by the establishment of the JBC shows that even candidates for judicial
Constitution in the President. The presidential power of positions at any level backed by people influential with the
appointment is dealt with in Sections 14, 15 and 16 of the Article. President could not always be assured of being recommended
for the consideration of the President, because they first had to
Article VIII is dedicated to the Judicial Department and defines the undergo the vetting of the JBC and pass muster there. Indeed,
duties and qualifications of Members of the Supreme Court, among the creation of the JBC was precisely intended to de-politicize the
others. Section 4(1) and Section 9 of this Article are the provisions Judiciary by doing away with the intervention of the Commission
specifically providing for the appointment of Supreme Court on Appointments.
Justices. In particular, Section 9 states that the appointment of
Supreme Court Justices can only be made by the President upon xxx
the submission of a list of at least three nominees by the JBC;
Section 4(1) of the Article mandates the President to fill the Fourth. Of the 23 sections in Article VII, three (i.e., Section 14,
vacancy within 90 days from the occurrence of the vacancy. Section15, and Section 16) concern the appointing powers of the
President.
Had the framers intended to extend the prohibition contained in
Section 15, Article VII to the appointment of Members of the Section 14 speaks of the power of the succeeding President to
Supreme Court, they could have explicitly done so. They could not revoke appointments made by an Acting President, and evidently
have ignored the meticulous ordering of the provisions. They would refers only to appointments in the Executive Department. It has
have easily and surely written the prohibition made explicit in no application to appointments in the Judiciary, because
Section 15, Article VII as being equally applicable to the temporary or acting appointments can only undermine the
appointment of Members of the Supreme Court in Article VIII itself, independence of the Judiciary due to their being revocable at
most likely in Section 4 (1), Article VIII. That such specification was will. The letter and spirit of the Constitution safeguard that
not done only reveals that the prohibition against the President or independence. Also, there is no law in the books that authorizes
Acting President making appointments within two months before the revocation of appointments in the Judiciary. Prior to their
the next presidential elections and up to the end of the Presidents mandatory retirement or resignation, judges of the first and
second level courts and the Justices of the third level courts may
P a g e 3 | 19
POLITICAL LAW REVIEW: IV. Executive Department Case Digests | Wednesday Class @ 4:30-9:30PM

only be removed for cause, but the Members of the Supreme Court appointed by the Philippine President. In every case, the
may be removed only by impeachment. appointing authority was the Chief Executive.

Section 16 covers only the presidential appointments that require The use of the generic term Members of the Supreme Court
confirmation by the Commission on Appointments. Thereby, the under Section 9, Article VIII in delineating the appointing
Constitutional Commission restored the requirement of authority under the 1987 Constitution, is not new. This was the
confirmation by the Commission on Appointments after the term used in the present line of Philippine Constitutions, from
requirement was removed from the 1973 Constitution. Yet, 1935 to 1987, and the inclusion of the Chief Justice with the
because of Section 9 of Article VIII, the restored requirement did general term Member of the Court has never been in doubt. In
not include appointments to the Judiciary. fact, Section 4(1) of the present Constitution itself confirms that
the Chief Justice is a Member of the Court when it provides that
Section 14, Section 15, and Section 16 are obviously of the same the Court may sit en banc or, in its discretion, in divisions of
character, in that they affect the power of the President to appoint. three, five, or seven Members. The Chief Justice is a Member of
The fact that Section 14 and Section 16 refer only to appointments the En Banc and of the First Division in fact, he is the Chair of the
within the Executive Department renders conclusive that Section En Banc and of the First Division but even as Chair is counted in
15 also applies only to the Executive Department. This conclusion the total membership of the En Banc or the Division for all
is consistent with the rule that every part of the statute must be purposes, particularly of quorum. Thus, at the same time that
interpreted with reference to the context, i.e. that every part must Section 4(1) speaks of a Supreme Court. . . composed of one
be considered together with the other parts, and kept subservient Chief Justice and fourteen Associate Justices, it likewise calls all
to the general intent of the whole enactment. It is absurd to of them Members in defining how they will sit in the Court.
assume that the framers deliberately situated Section 15 between
Section 14 and Section 16, if they intended Section 15 to cover all Thus, both by law and history, the Chief Justice has always been
kinds of presidential appointments. If that was their intention in a Member of the Court although, as a primus inter pares
respect of appointments to the Judiciary, the framers, if only to be appointed by the President together with every other Associate
clear, would have easily and surely inserted a similar prohibition in Justice.
Article VIII, most likely within Section 4 (1) thereof.
The Valenzuela decision gives the full flavor of how the election
Wherefore: ban issue arose because of Chief Justice Narvasas very candid
treatment of the facts and the issue. Valenzuela openly stated
Grants the petition in A.M. No. 10-2-5-SC and, accordingly, directs that at the root of the dispute was the then existing vacancy in
the Judicial and Bar Council: the Court and the difference of opinion on the matter between the
Executive and the Court on the application of Section 15, Article
(a) To resume its proceedings for the nomination of candidates to VII, in relation with Section 4(1) and 9 of Article VIII, of the
fill the vacancy to be created by the compulsory retirement of Chief Constitution.
Justice Reynato S. Puno by May 17, 2010;
What appears very clear from the decision, however, is that the
(b) To prepare the short list of nominees for the position of Chief factual situation the Court ruled upon, in the exercise of its
Justice; supervision of court personnel, was the appointment by the
President of two RTC judges during the period of the ban. It is
(c) To submit to the incumbent President the short list of nominees clear from the decision, too, that no immediate appointment was
for the position of Chief Justice on or before May 17, 2010; and ever made to the Court for the replacement of retired Justice
Ricardo Francisco as the JBC failed to meet on the required
(d) To continue its proceedings for the nomination of candidates to nominations prior to the onset of the election ban.
fill other vacancies in the Judiciary and submit to the President the
short list of nominees corresponding thereto in accordance with From this perspective, it appears clear to me that Valenzuela
this decision. should be read and appreciated for what it is a ruling made on
the basis of the Courts supervision over judicial personnel that
SO ORDERED. upholds the election ban as against the appointment of lower
court judges appointed pursuant to the period provided by
SEPARATE OPINION Section 9 of Article VIII. Thus, Valenzuelas application to the
filling up of a vacancy in the Supreme Court is a mere obiter
BRION, J: dictum as the Court is largely governed by Section 4(1) with
respect to the period of appointment. The Section 4(1) period, of
“I AGREE with the conclusion that the President can appoint the course and as already mentioned above, has an impact uniquely
Chief Justice and Members of the Supreme Court two months its own that is different from that created by the period provided
before a presidential election up to the end of the Presidents term, for the lower court under Section 9.
but DISAGREE with the conclusion that the authority to appoint
extends to the whole Judiciary.”
6. Velicaria-Garafil vs Office of the President
History tells us that, without exception, the Chief Justice of the VELICARIA – GARAFIL vs. OFFICE OF THE
Supreme Court has always been appointed by the head of the PRESIDENT
Executive Department. The only difference in their respective
appointments is the sovereignty under which they were appointed.
FACTS:
The Chief Justices under the American regime were appointed by  The present consolidated cases involve 4 petitions with
the President of the United States; one Chief Justice each was Atty. Velicaria-Garafil, who was appointed State Solicitor II
appointed under the Commonwealth and under the Japanese at the Office of the Solicitor General (OSG); with Atty.
Military Administration; and thereafter all the Chief Justices were Venturanza, who was appointed Prosecutor IV (City
Prosecutor) of Quezon City; with Villanueva, who was
P a g e 4 | 19
POLITICAL LAW REVIEW: IV. Executive Department Case Digests | Wednesday Class @ 4:30-9:30PM

appointed Administrator for Visayas of the Board of the appointment by the appointee who possesses all the
Administrators of the Cooperative Development Authority qualifications and none of the disqualifications. The
(CDA), and Francisca B. Rosquita, who was appointed concurrence of all these elements should always apply,
Commissioner of the National Commission of Indigenous regardless of when the appointment is made, whether
Peoples (NCIP); and with Atty. Tamondong, who was outside, just before, or during the appointment ban. These
appointed member of the Board of Directors of the Subic steps in the appointment process should always concur
Bay Metropolitan Authority (SBMA) and operate as a single process. There is no valid
appointment if the process lacks even one step. And,
 All petitions question the constitutionality of Executive unlike the dissent's proposal, there is no need to further
Order No. 2 (EO 2) for being inconsistent with Section 15, distinguish between an effective and an ineffective
Article VII of the 1987 Constitution appointment when an appointment is valid.

 Prior to the conduct of the May 2010 elections, then  Paragraph (b ), Section 1 of EO 2 considered as
PGMA issued more than 800 appointments to various midnight appointments those appointments to offices that
positions in several government offices will only be vacant on or after 11 March 2010 even though
the appointments are made prior to 11 March 2010. EO 2
 For purposes of the 2010 elections, March 10 2010 was remained faithful to the intent of Section 15, Article VII of
the cutoff date for valid appointments and the next day, 11 the 1987 Constitution: the outgoing President is prevented
March 2010, was the start of the ban on midnight from continuing to rule the country indirectly after the end
appointments. of his term.

 Section 15, Article VII of the 1987 Constitution  Acceptance is indispensable to complete an
recognizes as an exception to the ban on midnight appointment. Assuming office and taking the oath amount
appointments only "temporary appointments to executive to acceptance of the appointment.60 An oath of office is a
positions when continued vacancies therein will prejudice qualifying requirement for a public office, a prerequisite to
public service or endanger public safety." the full investiture of the office.

 None of the petitioners claim that their appointments fall Excluding the act of acceptance from the appointment
under this exception. process leads us to the very evil which we seek to avoid
(i.e., antedating of appointments). Excluding the act of
 On 30 June 2010, President Benigno S. Aquino III acceptance will only provide more occasions to honor the
(President Aquino) took his oath of office as President of Constitutional provision in the breach. The inclusion of
the Republic of the Philippines. acceptance by the appointee as an integral part of the
entire appointment process prevents the abuse of the
 On 30 July 2010, President Aquino issued EO 2 recalling, Presidential power to appoint. It is relatively easy to
withdrawing, and revoking appointments issued by PGMA antedate appointment papers and make it appear that
which violated the constitutional ban on midnight they were issued prior to the appointment ban, but it is
appointments. more difficult to simulate the entire appointment process
up until acceptance by the appointee.
ISSUE(s):
W/N the appointments of the petitioners and intervenor  Petitioners have failed to show compliance with all four
were midnight appointments within the coverage of EO 2? elements of a valid appointment. They cannot prove with
certainty that their appointment papers were transmitted
W/N all midnight appointments, including those of before the appointment ban took effect. On the other
petitioners and intervenors, were invalid? hand, petitioners admit that they took their oaths of office
during the appointment ban.
RULING:
 All of petitioners' appointments are midnight  Petitioners have failed to raise any valid ground for the
appointments and are void for violation of Section 15, Court to declare EO 2, or any part of it, unconstitutional.
Article VII of the 1987 Constitution. EO 2 is constitutional. Consequently, EO 2 remains valid and constitutional.

 Based on prevailing jurisprudence, appointment to a 7. Resident Marine Mammal vs Reyes


government post is a process that takes several steps to Resident Marine Mammals of the Protected Seascape Tanon
complete. Any valid appointment, including one made under Strait vs. Reyes GR 180771 and 181527 April 21, 2015
the exception provided in Section 15, Article VII of the 1987
Constitution, must consist of the President signing an Facts: Two sets of petitioners filed separate cases challenging
appointee's appointment paper to a vacant office, the the legality of Service Contract No. 46 (SC-46) awarded to Japan
official transmittal of the appointment paper (preferably Petroleum Exploration Co. (JAPEX).   The service contract
through the MRO), receipt of the appointment paper by the allowed JAPEX to conduct oil exploration in the Tañon Strait
appointee, and acceptance of the appointment by the during which it performed seismic surveys and drilled one
appointee evidenced by his or her oath of office or his or exploration well.  The first petition was brought on behalf of
her assumption to office. resident marine mammals in the Tañon Strait by two individuals
 The following elements should always concur in the acting as legal guardians and stewards of the marine mammals. 
making of a valid (which should be understood as both The second petition was filed by a non-governmental
complete and effective) appointment: (1) authority to organization representing the interests of fisherfolk, along with
appoint and evidence of the exercise of the authority; (2) individual representatives from fishing communities impacted by
transmittal of the appointment paper and evidence of the the oil exploration activities. The petitioners filed their cases in
transmittal; (3) a vacant position at the time of appointment; 2007, shortly after JAPEX began drilling in the strait.  Also
and (4) receipt of the appointment paper and acceptance of impleaded as an unwilling co-petitioner is former President Gloria
P a g e 5 | 19
POLITICAL LAW REVIEW: IV. Executive Department Case Digests | Wednesday Class @ 4:30-9:30PM

Macapagal-Arroyo, for her express declaration and undertaking in


the ASEAN Charter to protect the Tañon Strait, among others. Held: Under the Rules of Court when the consent of a party who
should be joined as a plaintiff cannot be obtained, he or she may
In 2008, JAPEX and the government of the Philippines mutually be made a party defendant to the case. This will put the unwilling
terminated the service contract and oil exploration activities party under the jurisdiction of the Court, which can properly
ceased.  The Supreme Court consolidated the cases for the implead him or her through its processes. The unwilling party's
purpose of review. Due to the alleged grave constitutional name cannot be simply included in a petition, without his or her
violations and paramount public interest in the case, not to mention knowledge and consent, as such would be a denial of due
the fact that the actions complained of could be repeated, the process.
Court found it necessary to reach the merits of the case even Moreover, the reason cited by the petitioners Stewards for
though the particular service contract had been terminated.  including former President Macapagal-Arroyo in their petition, is
not sufficient to implead her as an unwilling co-petitioner.
Impleading the former President as an unwilling co-
1. Issue: WON the SC-46 entered into and signed by the petitioner, for an act she made in the performance of the
Secretary of DOE is considered as the act of PGMA under the functions of her office, is contrary to the public policy
“alter ego principle” NO against embroiling the President in suits, "to assure the
exercise of Presidential duties and functions free from any
Held: Paragraph 4, Section 2, Article XII of the 1987 Constitution hindrance or distraction, considering that being the Chief
requires that the President himself enter into any service contract Executive of the Government is a job that, aside from
for the exploration of petroleum. requiring all of the office holder's time, also demands
undivided attention."59
SC-46 appeared to have been entered into and signed only by the
DOE through its then Secretary, Vicente S. Perez, Jr., contrary to Therefore, former President Macapagal-Arroyo cannot be
the said constitutional requirement. Moreover, public respondents impleaded as one of the petitioners in this suit. Thus, her name is
have neither shown nor alleged that Congress was subsequently stricken off the title of this case.
notified of the execution of such contract. Public respondents’
implied argument that based on the “alter ego principle,” their acts 8. Almario vs Executive Secretary
are also that of then President Macapagal-Arroyo’s, CANNOT National Artist for Literature Virgilio Almario, et al. vs. The
apply in this case. Executive Secretary, 701 SCRA 269, G.R. No. 189028 July 16,
2013
In Joson v. Torres, 290 SCRA 279 (1998), we explained the
LEONARDO-DE CASTRO, J.:
concept of the alter ego principle or the doctrine of qualified
political agency and its limit in this wise: History of the Order of National Artists
On April 3, 1992, Republic Act No. 7356, otherwise known as the
Under this doctrine, which recognizes the establishment
Law Creating the National Commission for Culture and the Arts,
of a single executive, all executive and administrative
was signed into law. It established the National Commission for
organizations are adjuncts of the Executive Department,
Culture and the Arts (NCCA) and gave it an extensive mandate
the heads of the various executive departments are
over the development, promotion and preservation of the Filipino
assistants and agents of the Chief Executive, and, except
national culture and arts and the Filipino cultural heritage.
in cases where the Chief Executive is required by the
Constitution or law to act in person or the exigencies of _____
the situation demand that he act personally, the FACTS: On January 30, 2007, a joint meeting of the NCCA
multifarious executive and administrative functions of the Board of Commissioners and the CCP Board of Trustees was
Chief Executive are performed by and through the held to discuss, among others, the evaluation of the 2009 Order
executive departments, and the acts of the Secretaries of of National Artists and the convening of the National Artist Award
such departments, performed and promulgated in the Secretariat. The nomination period was set for September 2007
regular course of business, are, unless disapproved or to December 31, 2007, which was later extended to February 28,
reprobated by the Chief Executive presumptively the acts 2008. The pre-screening of nominations was held from January
of the Chief Executive. to March 2008.
In this case, the public respondents have failed to show that On April 3, 2009, the First Deliberation Panel met. A total of 87
the President had any participation in SC-46. Their argument nominees were considered during the deliberation and a
that their acts are actually the acts of then President preliminary shortlist of 32 names was compiled.
Macapagal-Arroyo, absent proof of her disapproval, must fail On April 23, 2009, the Second Deliberation Panel purportedly
as the requirement that the President herself enter into these composed of an entirely new set of Council of Experts met and
kinds of contracts is embodied not just in any ordinary shortlisted 13 out of the 32 names in the preliminary shortlist. On
statute, but in the Constitution itself. These service contracts May 6, 2009, the final deliberation was conducted by the 30-
involving the exploitation, development, and utilization of our member Final Deliberation Panel comprised of the CCP Board of
natural resources are of paramount interest to the present and Trustees and the NCCA Board of Commissioners and the living
future generations. Hence, safeguards were put in place to insure National Artists. From the 13 names in the second shortlist, a
that the guidelines set by law are meticulously observed and final list of four names was agreed upon.
likewise to eradicate the corruption that may easily penetrate
departments and agencies by ensuring that the President has On May 6, 2009, a letter, signed jointly by the Chairperson of the
authorized or approved of these service contracts herself. NCCA, Undersecretary Vilma Labrador, and the President and
Artistic Director of the CCP, Mr. Nestor Jardin, was sent to the
President: submitting a recommendation of the NCCA Board of
2. Issue: WON impleading PGMA as an unwilling co-petitioner is Trustees and CCP Board of Trustees for the Proclamation of the
contrary to the public policy against embroiling the President in following as 2009 Order of National Artists:
suits. YES
P a g e 6 | 19
POLITICAL LAW REVIEW: IV. Executive Department Case Digests | Wednesday Class @ 4:30-9:30PM

1. Mr. MANUEL CONDE+ (Posthumous) – Film and the CCP Boards without clearly indicating the basis thereof.
and Broadcast Arts For petitioners, the President’s discretion to name National
Artists cannot be exercised to defeat the recommendations made
2. Dr. RAMON SANTOS – Music
by the CCP and NCCA Boards after a long and rigorous
3. Mr. LAZARO FRANCISCO+ (Posthumous) – screening process and with the benefit of expertise and
Literature experience. The addition of four names to the final list submitted
by the Boards of the CCP and the NCCA and the deletion of one
4. Mr. FEDERICO AGUILAR-ALCUAZ – Visual
name from the said list constituted a substitution of judgment by
Arts
the President and a unilateral reconsideration without clear
According to respondents, the aforementioned letter was referred justification of the decision of the First, Second and Final
by the Office of the President to the Committee on Honors. Deliberation Panels composed of experts.
Meanwhile, the Office of the President allegedly received
Petitioners further argue that the choice of respondent Guidote
nominations from various sectors, cultural groups and individuals
Alvarez was illegal and unethical because, as the then Executive
strongly endorsing private respondents Cecile Guidote-Alvarez,
Director of the NCCA and presidential adviser on culture and
Carlo Magno Jose Caparas, Francisco Mañosa and Jose Moreno.
arts, she was disqualified from even being nominated. Moreover,
The Committee on Honors purportedly processed these
such action on the part of the former President constituted grave
nominations and invited resource persons to validate the
abuse of discretion as it gave preferential treatment to
qualifications and credentials of the nominees.
respondent Guidote-Alvarez by naming the latter a National Artist
The Committee on Honors thereafter submitted a memorandum to despite her not having been nominated and, thus, not subjected
then President Gloria Macapagal-Arroyo recommending the to the screening process provided by the rules for selection to the
conferment of the Order of National Artists on the four Order of National Artists. Her inclusion in the list by the President
recommendees of the NCCA and the CCP Boards, as well as on represented a clear and manifest favor given by the President in
private respondents Guidote-Alvarez, Caparas, Mañosa and that she was exempted from the process that all other artists
Moreno. Acting on this recommendation, Proclamation No. 1823 have to undergo. According to petitioners, it may be said that the
declaring Manuel Conde a National Artist was issued on June 30, President used a different procedure to qualify respondent
2009. Subsequently, on July 6, 2009, Proclamation Nos. 1824 to Guidote-Alvarez. This was clearly grave abuse of discretion for
1829 were issued declaring Lazaro Francisco, Federico Aguilar being manifest and undue bias violative of the equal protection
Alcuaz and private respondents Guidote-Alvarez, Caparas, clause.
Mañosa and Moreno, respectively, as National Artists. This was
Respondent Caparas refutes the contention of the petitioning
subsequently announced to the public by then Executive Secretary
National Artists and insists that there could be no prejudice to the
Eduardo Ermita on July 29, 2009.
latter. They remain to be National Artists and continue to receive
Convinced that, by law, it is the exclusive province of the NCCA the emoluments, benefits and other privileges pertaining to them
Board of Commissioners and the CCP Board of Trustees to select by virtue of that honor. On the other hand, all the other
those who will be conferred the Order of National Artists and to set petitioners failed to show any material and personal injury or
the standard for entry into that select group, petitioners instituted harm caused to them by the conferment of the Order of National
this petition for prohibition, certiorari and injunction (with prayer for Artists on respondents Guidote-Alvarez, Caparas, Mañosa and
restraining order) praying that the Order of National Artists be Moreno. The rule on standing may not be relaxed in favor of the
conferred on Dr. Santos and that the conferment of the Order of petitioners as no question of constitutionality has been raised
National Artists on respondents Guidote-Alvarez, Caparas, and no issue of transcendental importance is involved.
Mañosa and Moreno be enjoined and declared to have been
On the merits, respondent Caparas contends that no grave
rendered in grave abuse of discretion.
abuse of discretion attended his proclamation as National Artist.
In a Resolution dated August 25, 2009, the Court issued a status The former President considered the respective
quo order enjoining "public respondents" "from conferring the rank recommendations of the NCCA and the CCP Boards and of the
and title of the Order of National Artists on private respondents; Committee on Honors in eventually declaring him (Caparas) as
from releasing the cash awards that accompany such conferment National Artist. The function of the NCCA and the CCP Boards is
and recognition; and from holding the acknowledgment simply to advise the President. The award of the Order of
ceremonies for recognition of the private respondents as National National Artists is the exclusive prerogative of the President who
Artists." is not bound in any way by the recommendation of the NCCA
and the CCP Boards. The implementing rules and regulations or
guidelines of the NCCA cannot restrict or limit the exclusive
Contention of the Parties power of the President to select the recipients of the Order of
All of the petitioners claim that former President Macapagal-Arroyo National Artists.
gravely abused her discretion in disregarding the results of the On the other hand, the original position of the Office of the
rigorous screening and selection process for the Order of National Solicitor General (OSG) was similar to that of respondent
Artists and in substituting her own choice for those of the Caparas. In a subsequent manifestation, however, the OSG
Deliberation Panels. According to petitioners, the President’s stated that the current Board of Commissioners of the NCCA
discretion to name National Artists is not absolute but limited. In agree with the petitioners that the President cannot honor as a
particular, her discretion on the matter cannot be exercised in the National Artist one who was not recommended by the joint
absence of or against the recommendation of the NCCA and the Boards of the NCCA and the CCP. The implementing rules and
CCP. In adding the names of respondents Caparas, Guidote- regulations of Executive Order No. 236, s. 2003, recognized the
Alvarez, Mañosa and Moreno while dropping Dr. Santos from the binding character of the recommendation of the NCCA and the
list of conferees, the President’s own choices constituted the CCP Boards and limited the authority of the Committee on
majority of the awardees in utter disregard of the choices of the Honors to the determination that (1) there has been no grave
NCCA and the CCP and the arts and culture community which abuse of discretion on the part of the NCCA and the CCP Boards
were arrived at after a long and rigorous process of screening and in making the nomination, and (2) the nominee is in good
deliberation. Moreover, the name of Dr. Santos as National Artist standing. Where a nomination meets the said two criteria, a
for Music was deleted from the final list submitted by the NCCA
P a g e 7 | 19
POLITICAL LAW REVIEW: IV. Executive Department Case Digests | Wednesday Class @ 4:30-9:30PM

recommendation to the President to confer the award shall be to respondents Guidote-Alvarez, Caparas, Mañosa and Moreno;
made. The conferment of the Order of National Artists on said
respondents was therefore made with grave abuse of discretion
The OSG further argued that, while the President exercises control
and should be set aside.—There was a violation of the equal
over the NCCA and the CCP, the President has the duty to
protection clause of the Constitution when the former President
faithfully execute the laws, including the NCCA-CCP guidelines for
gave preferential treatment to respondents Guidote-Alvarez,
selection of National Artists and the implementing rules of
Caparas, Mañosa and Moreno. The former President’s
Executive Order No. 236, s. 2003. Moreover, the laws recognize
constitutional duty to faithfully execute the laws and observe the
the expertise of the NCCA and the CCP in the arts and tasked
rules, guidelines and policies of the NCCA and the CCP as to the
them to screen and select the artists to be conferred the Order of
selection of the nominees for conferment of the Order of National
National Artists. Their mandate is clear and exclusive as no other
Artists proscribed her from having a free and uninhibited hand in
agency possesses such expertise.
the conferment of the said award. The manifest disregard of the
ISSUE: Whether the proclamation by the President of respondents rules, guidelines and processes of the NCCA and the CCP was
Guidote-Alvarez, Caparas, Mañosa and Moreno, as a National an arbitrary act that unduly favored respondents Guidote-
Artists is invalid on the ground that they were not recommended by Alvarez, Caparas, Mañosa and Moreno. The conferment of the
the joint Boards of the NCCA and the CCP Order of National Artists on said respondents was therefore
made with grave abuse of discretion and should be set aside.
HELD: YES.
While the Court invalidates today the proclamation of
In view of the various stages of deliberation in the selection respondents Guidote-Alvarez, Caparas, Mañosa and Moreno as
process and as a consequence of his/her duty to faithfully enforce National Artists, such action should not be taken as a
the relevant laws, the discretion of the President in the matter of pronouncement on whether they are worthy to be conferred that
the Order of National Artists is confined to the names submitted to honor. Only the President, upon the advise of the NCCA and the
him/her by the NCCA and the CCP Boards. This means that the CCP Boards, may determine that. The Court simply declares
President could not have considered conferment of the Order that, as the former President committed grave abuse of
of National Artists on any person not considered and discretion in issuing Proclamation Nos. 1826 to 1829 dated July
recommended by the NCCA and the CCP Boards. That is the 6, 2009, the said proclamations are invalid. However, nothing in
proper import of the provision of Executive Order No. 435, s. 2005, this Decision should be read as a disqualification on the part of
that the NCCA and the CCP “shall advise the President on the respondents Guidote-Alvarez, Caparas, Mañosa and Moreno to
conferment of the Order of National Artists.” Applying this to the be considered for the honor of National Artist in the future,
instant case, the former President could not have properly subject to compliance with the laws, rules and regulations
considered respondents Guidote-Alvarez, Caparas, Mañosa and governing said award.
Moreno, as their names were not recommended by the NCCA and
the CCP Boards. Otherwise, not only will the stringent selection 9. Monsanto vs Factoran
and meticulous screening process be rendered futile, the Monsanto vs. Factoran
respective mandates of the NCCA and the CCP Board of Trustees
under relevant laws to administer the conferment of Order of Facts: Salvacion Monsanto (then assistant treasurer of Calbayog
National Artists, draft the rules and regulations to guide its City) and three other accused were convicted of complex crime
deliberations, formulate and implement policies and plans, and of estafa thru falsification of public documents. Petitioner
undertake any and all necessary measures in that regard will also Monsanto appealed her conviction to this Court which
become meaningless. subsequently affirmed the same. She then filed a motion for
reconsideration but while said motion was pending, she was
In the matter of the conferment of the Order of National Artists, the extended on December 17, 1984 by then President Marcos
President may or may not adopt the recommendation or advice of absolute pardon which she accepted on December 21, 1984. By
the NCCA and the CCP Boards. In other words, the advice of the reason of said pardon, petitioner wrote the Calbayog City
NCCA and the CCP is subject to the President’s discretion. treasurer requesting that she be restored to her former post as
Nevertheless, the President’s discretion on the matter is not totally assistant city treasurer since the same was still vacant.
unfettered, nor the role of the NCCA and the CCP Boards
meaningless. Discretion is not a free-spirited stallion that runs and Petitioner's letter-request was referred to the Ministry of Finance
roams wherever it pleases but is reined in to keep it from straying. for resolution in view of the provision of the Local Government
In its classic formulation, “discretion is not unconfined and vagrant” Code transferring the power of appointment of treasurers from
but “canalized within banks that keep it from overflowing.” The the city governments to the said Ministry. The Finance Ministry
President’s power must be exercised in accordance with existing ruled that petitioner may be reinstated to her position without the
laws. Section 17, Article VII of the Constitution prescribes faithful necessity of a new appointment not earlier than the date she was
execution of the laws by the President: Sec. 17. The President extended the absolute pardon. Seeking reconsideration of the
shall have control of all the executive departments, bureaus and foregoing ruling, petitioner wrote the Ministry on April 17, 1985
offices. He shall ensure that the laws be faithfully executed. stressing that the full pardon bestowed on her has wiped out the
(Emphasis supplied.) The President’s discretion in the conferment crime which implies that her service in the government has never
of the Order of National Artists should be exercised in accordance been interrupted and therefore the date of her reinstatement
with the duty to faithfully execute the relevant laws. The faithful should correspond to the date of her preventive suspension
execution clause is best construed as an obligation imposed on which is August 1, 1982; that she is entitled to backpay for the
the President, not a separate grant of power. It simply underscores entire period of her suspension; and that she should not be
the rule of law and, corollarily, the cardinal principle that the required to pay the proportionate share of the amount of
President is not above the laws but is obliged to obey and execute P4,892.50. The Ministry of Finance, however, referred
them. This is precisely why the law provides that “[a]dministrative petitioner's letter to the Office of the President for further review
or executive acts, orders and regulations shall be valid only when and action. On April 15, 1986, said Office, through Deputy
they are not contrary to the laws or the Constitution.” Executive Secretary Fulgenio S. Factoran, Jr. held that acquittal,
not absolute pardon, of a former public officer is the only ground
There was a violation of the equal protection clause of the for reinstatement to his former position and entitlement to
Constitution when the former President gave preferential treatment
P a g e 8 | 19
POLITICAL LAW REVIEW: IV. Executive Department Case Digests | Wednesday Class @ 4:30-9:30PM

payment of his salaries, benefits and emoluments due to him


during the period of his suspension pendente lite. In 2012, Estrada once more ventured into the political arena, and
filed a Certificate of Candidacy, this time vying for a local elective
Issue: Whether or not the grant of pardon removes the absolute post, that of the Mayor of the City of Manila.
disqualification or ineligibility from public office of those convicted
of estafa thru falsification of public documents. Petitioner Risos-Vidal filed a Petition for Disqualification against
Estrada before the Comelec stating that Estrada is disqualified to
Ruling: Yes but regaining the former post is subject to the usual run for public office because of his conviction for plunder
procedure required for new appointment. sentencing him to suffer the penalty of reclusion perpetua with
perpetual absolute disqualification. Petitioner relied on Section
The Court held that pardon may remit all the penal consequences 40 of the Local Government Code (LGC), in relation to Section
of a criminal indictment if only to give meaning to the fiat that a 12 of the Omnibus Election Code (OEC). 
pardon, being a presidential prerogative, should not be
circumscribed by legislative action, it is a fictitious belief that The Comelec dismissed the petition for disqualification holding
pardon blots out the guilt of an individual and that once he is that President Estrada’s right to seek public office has been
absolved, he should be treated as if he were innocent. Pardon effectively restored by the pardon vested upon him by former
cannot mask the acts constituting the crime. President Gloria M. Arroyo.

Thus, notwithstanding the expansive and effusive language of Estrada won the mayoralty race in May 13, 2013 elections.
the Garland case, we are in full agreement with the commonly-held Alfredo Lim, who garnered the second highest votes, intervened
opinion that pardon does not ipso facto  restore a convicted felon to and sought to disqualify Estrada for the same ground as the
public office necessarily relinquished or forfeited by reason of the contention of Risos-Vidal and praying that he be proclaimed as
conviction although such pardon undoubtedly restores his eligibility Mayor of Manila.
for appointment to that office.
Issue:
The rationale is plainly evident Public offices are intended primarily
for the collective protection, safety and benefit of the common May former President Joseph Estrada run for public office
good. They cannot be compromised to favor private interests. To despite having been convicted of the crime of plunder which
insist on automatic reinstatement because of a mistaken notion carried an accessory penalty of perpetual disqualification to hold
that the pardon virtually acquitted one from the offense of estafa public office?
would be grossly untenable. A pardon, albeit full and plenary,
cannot preclude the appointing power from refusing appointment Held:
to anyone deemed to be of bad character, a poor moral risk, or
who is unsuitable by reason of the pardoned conviction. Yes. Estrada was granted an absolute pardon that fully restored
all his civil and political rights, which naturally includes the right to
For petitioner Monsanto, this is the bottom line: the absolute seek public elective office, the focal point of this controversy. The
disqualification or ineligibility from public office forms part of the wording of the pardon extended to former President Estrada is
punishment prescribed by the Revised Penal Code for estafa thru complete, unambiguous, and unqualified. It is likewise unfettered
falsification of public documents. It is clear from the authorities by Articles 36 and 41 of the Revised Penal Code. The only
referred to that when her guilt and punishment were expunged by reasonable, objective, and constitutional interpretation of the
her pardon, this particular disability was likewise removed. language of the pardon is that the same in fact conforms to
Henceforth, petitioner may apply for reappointment to the office Articles 36 and 41 of the Revised Penal Code. 
which was forfeited by reason of her conviction. And in considering
her qualifications and suitability for the public post, the facts It is insisted that, since a textual examination of the pardon given
constituting her offense must be and should be evaluated and to and accepted by former President Estrada does not actually
taken into account to determine ultimately whether she can once specify which political right is restored, it could be inferred that
again be entrusted with public funds. Stated differently, the pardon former President Arroyo did not deliberately intend to restore
granted to petitioner has resulted in removing her disqualification former President Estrada’s rights of suffrage and to hold public
from holding public employment but it cannot go beyond that. To office, orto otherwise remit the penalty of perpetual absolute
regain her former post as assistant city treasurer, she must re- disqualification. Even if her intention was the contrary, the same
apply and undergo the usual procedure required for a new cannot be upheld based on the pardon’s text.
appointment.
The pardoning power of the President cannot be limited by
10. Risos-Vidal vs COMELEC legislative action.
Risos-Vidal vs. COMELEC
The 1987 Constitution, specifically Section 19 of Article VII and
Facts: Section 5 of Article IX-C, provides that the President of the
In September 12, 2007, the Sandiganbayan convicted former Philippines possesses the power to grant pardons, along with
President Estrada for the crime of plunder and was sentenced to other acts of executive clemency, to wit:
suffer the penalty of Reclusion Perpetua and the accessory
penalties of civil interdiction during the period of sentence and Section 19. Except in cases of impeachment, or as otherwise
perpetual absolute disqualification. On October 25, 2007, however, provided in this Constitution, the President may grant reprieves,
former President Gloria Macapagal Arroyo extended executive commutations, and pardons, and remit fines and forfeitures, after
clemency, by way of pardon, to former President Estrada, explicitly conviction by final judgment.
stating that he is restored to his civil and political rights.

In 2009, Estrada filed a Certificate of Candidacy for the position of He shall also have the power to grant amnesty with the
President. None of the disqualification cases against him concurrence of a majority of all the Members of the Congress.
prospered but he only placed second in the results.
P a g e 9 | 19
POLITICAL LAW REVIEW: IV. Executive Department Case Digests | Wednesday Class @ 4:30-9:30PM

effectivity contingent upon the fulfilment of the aforementioned


commitment nor to limit the scope of the pardon.
xxxx
Besides, a preamble is really not an integral part of a law. It is
merely an introduction to show its intent or purposes. It cannot be
Section 5. No pardon, amnesty, parole, or suspension of sentence the origin of rights and obligations. Where the meaning of a
for violation of election laws, rules, and regulations shall be statute is clear and unambiguous, the preamble can neither
granted by the President without the favorable recommendation of expand nor restrict its operation much less prevail over its text.
the Commission.
If former President Arroyo intended for the pardon to be
It is apparent from the foregoing constitutional provisions that the conditional on Respondent’s promise never to seek a public
only instances in which the President may not extend pardon office again, the former ought to have explicitly stated the same
remain to be in: (1) impeachment cases; (2) cases that have not in the text of the pardon itself. Since former President Arroyo did
yet resulted in a final conviction; and (3) cases involving violations not make this an integral part of the decree of pardon, the
of election laws, rules and regulations in which there was no Commission is constrained to rule that the 3rd preambular clause
favorable recommendation coming from the COMELEC. Therefore, cannot be interpreted as a condition to the pardon extended to
it can be argued that any act of Congress by way of statute cannot former President Estrada.
operate to delimit the pardoning power of the President.

The proper interpretation of Articles 36 and 41 of the Revised 11. Kulayan vs Tan
Penal Code. KULAYAN vs. TAN (G.R. No. 187298 July 03, 2012)

A close scrutiny of the text of the pardon extended to former Facts: Three members from the International Committee of the
President Estrada shows that both the principal penalty of Red Cross (ICRC) were kidnapped in the vicinity of the Provincial
reclusion perpetua and its accessory penalties are included in the Capitol in Patikul, Sulu. Andres Notter, Eugenio Vagni, and Marie
pardon. The sentence which states that “(h)e is hereby restored to jean Lacaba were purportedly inspecting a water sanitation
his civil and political rights,” expressly remitted the accessory project for the Sulu Provincial Jail when they were seized by
penalties that attached to the principal penalty of reclusion three armed men who were later confirmed to be members of the
perpetua. Hence, even if we apply Articles 36 and 41 of the Abu Sayyaf Group (ASG). A Local Crisis Committee, later
Revised Penal Code, it is indubitable from the text of the pardon renamed Sulu Crisis Management Committee (Committee) was
that the accessory penalties of civil interdiction and perpetual then formed to investigate the kidnapping incident. The
absolute disqualification were expressly remitted together with the Committee convened under the leadership of respondent
principal penalty of reclusion perpetua. Abdusakur Mahail Tan, the Provincial Governor of Sulu.

The disqualification of former President Estrada under Governor Tan issued the Proclamation No. 1 Series of
Section 40 of the LGC in relation to Section 12 of the OEC was 2009, declaring a state of emergency in the province of Sulu. The
removed by his acceptance of the absolute pardon granted to Proclamation cited the kidnapping incident as a ground for the
him said declaration, describing it as terrorist act pursuant to the
Human Security act (R.A. 937). It also invoked Section 465 of the
While it may be apparent that the proscription in Section 40(a) of Local Government Code of 1991 (R.A 7160), which bestows on
the LGC is worded in absolute terms, Section 12 of the OEC the Provincial Governor the power to carry out emergency
provides a legal escape from the prohibition – a plenary pardon or measures during man-made and natural disasters and
amnesty. In other words, the latter provision allows any person calamities, and to call upon the appropriate national law
who has been granted plenary pardon or amnesty after conviction enforcement agencies to suppress disorder and lawless violence.
by final judgment of an offense involving moral turpitude, inter alia, In the Proclamation, Tan called upon the PNP and the Civilian
to run for and hold any public office, whether local or national Emergency Force (CEF) to set up checkpoints and chokepoints,
position. conduct general search and seizures including arrests, and other
necessary actions necessary to ensure public safety.
The third preambular clause of the pardon did not operate to
make the pardon conditional. Petitioners, Jamar Kulayan, et. Al claimed that
Proclamation No. 1-09 was issued ultra vires, and thus null and
Contrary to Risos-Vidal’s declaration, the third preambular clause void, for violating Sections 1 and 18, Article VII of the
of the pardon, i.e., "[w]hereas, Joseph Ejercito Estrada has publicly Constitution, which grants the President sole authority to
committed to no longer seek any elective position or office," neither exercise emergency powers and calling-out powers as the chief
makes the pardon conditional, nor militate against the conclusion executive of the Republic and commander-in-chief of the armed
that former President Estrada’s rights to suffrage and to seek forces.
public elective office have been restored.
Issue: Whether or not a governor can exercise the calling-out
This is especially true as the pardon itself does not explicitly powers of a President.
impose a condition or limitation, considering the unqualified use of
the term "civil and political rights" as being restored. Jurisprudence Held: It has already been established that there is one repository
educates that a preamble is not an essential part of an act as it is of executive powers, and that is the President of the Republic.
an introductory or preparatory clause that explains the reasons for This means that when Section 1, Article VII of the Constitution
the enactment, usually introduced by the word "whereas." speaks of executive power, it granted to the President and no
Whereas clauses do not form part of a statute because, strictly one else. Corollarily, it is only the President, as Executive, who is
speaking, they are not part of the operative language of the authorized to exercise emergency powers as provided under
statute. In this case, the whereas clause at issue is not an integral Section 23, Article VI, of the Constitution, as well as what
part of the decree of the pardon, and therefore, does not by itself became known as the calling-out powers under Section 7, Article
alone operate to make the pardon conditional or to make its VII thereof.
P a g e 10 | 19
POLITICAL LAW REVIEW: IV. Executive Department Case Digests | Wednesday Class @ 4:30-9:30PM

Constitution. The President gave the DILG Secretary the power


While the President is still a civilian, Article II, Section 3 of the to exercise, not merely administrative supervision, but control
Constitution mandates that civilian authority is, at all times, over the ARMM since the latter could suspend ARMM officials
supreme over the military, making the civilian president the and replace them. etitioner ARMM officials claimed that the
nation’s supreme military leader. The net effect of Article II, Section President had no factual basis for declaring a state of
3, when read with Article VII, Section 18, is that a civilian President emergency, especially in the Province of Sultan Kudarat and the
is the ceremonial, legal and administrative head of the armed City of Cotabato, where no critical violent incidents occurred.
forces. The Constitution does not require that the President must
be possessed of military training and talents, but as Commander- Office of the Solicitor General (OSG) insisted that the President
in-Chief, he has the power to direct military training and talents, but issued Proclamation 1946, not to deprive the ARMM of its
as Commander-in-Chief, he has the power to direct military autonomy, but to restore peace and order in subject places.8She
operations and to determine military strategy. Normally, he would issued the proclamation pursuant to her "calling out" power9 as
be expected to delegate the actual command of the armed forces Commander-in-Chief under the first sentence of Section 18,
to military experts, but the ultimate power is his. Article VII of the Constitution. The determination of the need to
exercise this power rests solely on her wisdom.10 She must use
Given the foregoing, Governor Tan is not endowed with the power her judgment based on intelligence reports and such best
to call upon the armed forces at his own bidding. In issuing the information as are available to her to call out the armed forces to
assailed proclamation, Governor Tan exceeded his authority when suppress and prevent lawless violence wherever and whenever
he declared state of emergency and called upon the Armed these reared their ugly heads. On the other hand, the President
Forces, the police, and his own Civilian Emergency Force. The merely delegated through AOs 273 and 273-A her supervisory
calling-out powers contemplated under the Constitution is powers over the ARMM to the DILG Secretary who was her alter
exclusive to the President. An exercise by another official, even if ego any way.
he is the local chief executive, is ultra vires, and may not be
justified by the invocation of Section 465 of the Local Government Issues:
Code. 1. Whether Proclamation 1946 and AOs 273 and 273-A violate
the principle of local autonomy under Section 16, Article X of the
12. Ampatuan vs Puno Constitution, and Section 1, Article V of the Expanded ARMM
G.R. No. 190259               June 7, 2011 Organic Act - No

DATU ZALDY UY AMPATUAN, ANSARUDDIN ADIONG, REGIE 2. Whether President Arroyo invalidly exercised emergency
SAHALI-GENERALE Petitioners,  powers when she called out the AFP and the PNP to prevent and
vs. suppress all incidents of lawless violence in Maguindanao,
HON. RONALDO PUNO, in his capacity as Secretary of the Sultan Kudarat, and Cotabato City - No
Department of Interior and Local Government and alter-ego of 3. Whether the President had factual bases for her actions - Yes
President Gloria Macapagal-Arroyo, and anyone acting in his
stead and on behalf of the President of the Philippines, Held:
ARMED FORCES OF THE PHILIPPINES (AFP), or any of their 1. No.
units operating in the Autonomous Region in Muslim The DILG Secretary did not take over control of the powers of the
Mindanao (ARMM), and PHILIPPINE NATIONAL POLICE, or ARMM. After law enforcement agents took respondent Governor
any of their units operating in ARMM, Respondents. of ARMM into custody for alleged complicity in the Maguindanao
massacre, the ARMM Vice-Governor, petitioner Ansaruddin
Facts: Adiong, assumed the vacated post on December 10, 2009
On November 24, 2009, the day after the gruesome massacre of pursuant to the rule on succession found in Article VII, Section
57 men and women, including some news reporters, then 12,14 of RA 9054. In turn, Acting Governor Adiong named the
President Gloria Macapagal-Arroyo issued Proclamation then Speaker of the ARMM Regional Assembly, petitioner
1946, placing "the Provinces of Maguindanao and Sultan Kudarat Sahali-Generale, Acting ARMM Vice-Governor.15 In short, the
and the City of Cotabato under a state of emergency." She DILG Secretary did not take over the administration or operations
directed the Armed Forces of the Philippines (AFP) and the of the ARMM.
Philippine National Police (PNP) "to undertake such measures as
may be allowed by the Constitution and by law to prevent and 2. No.
suppress all incidents of lawless violence" in the named places. Petitioners contend that the President unlawfully exercised
emergency powers when she ordered the deployment of AFP
Three days later or on November 27, President Arroyo also issued and PNP personnel in the places mentioned in the
Administrative Order 273 (AO 273)2"transferring" supervision of proclamation.16 But such deployment is not by itself an exercise
the Autonomous Region of Muslim Mindanao (ARMM) from the of emergency powers as understood under Section 23 (2), Article
Office of the President to the Department of Interior and Local VI of the Constitution, which provides:
Government (DILG). But, due to issues raised over the terminology
used in AO 273, the President issued Administrative Order 273-A SECTION 23. x x x (2) In times of war or other national
(AO 273-A) amending the former, by "delegating" instead of emergency, the Congress may, by law, authorize the President,
"transferring" supervision of the ARMM to the DILG. for a limited period and subject to such restrictions as it may
prescribe, to exercise powers necessary and proper to carry out
Claiming that the President’s issuances encroached on the a declared national policy. Unless sooner withdrawn by
ARMM’s autonomy, petitioners Datu Zaldy Uy Ampatuan, resolution of the Congress, such powers shall cease upon the
Ansaruddin Adiong, and Regie Sahali-Generale, all ARMM next adjournment thereof.
officials,4 filed this petition and alleged he proclamation and the
The President did not proclaim a national emergency, only a
orders empowered the DILG Secretary to take over ARMM’s
state of emergency in the three places mentioned. And she did
operations and seize the regional government’s powers, in
not act pursuant to any law enacted by Congress that authorized
violation of the principle of local autonomy under Republic Act
her to exercise extraordinary powers. The calling out of the
9054 (also known as the Expanded ARMM Act) and the
armed forces to prevent or suppress lawless violence in such
P a g e 11 | 19
POLITICAL LAW REVIEW: IV. Executive Department Case Digests | Wednesday Class @ 4:30-9:30PM

places is a power that the Constitution directly vests in the violence must be done swiftly and decisively if it were to have
President. She did not need a congressional authority to exercise any effect at all.
the same.
The President’s call on the armed forces to prevent or suppress
lawless violence springs from the power vested in her under 3. Yes.
Section 18, Article VII of the Constitution, which provides. The Ampatuan and Mangudadatu clans are prominent families
SECTION 18. The President shall be the Commander-in-Chief of engaged in the political control of Maguindanao. It is also a
all armed forces of the Philippines and whenever it becomes known fact that both families have an arsenal of armed followers
necessary, he may call out such armed forces to prevent or who hold elective positions in various parts of the ARMM and the
suppress lawless violence, invasion or rebellion. x x x rest of Mindanao.

While it is true that the Court may inquire into the factual bases for he Ampatuan forces are estimated to be approximately two
the President’s exercise of the above power,18 it would generally thousand four hundred (2,400) persons, equipped with about two
defer to her judgment on the matter. As the Court acknowledged in thousand (2,000) firearms, about four hundred (400) of which
Integrated Bar of the Philippines v. Hon. Zamora,19 it is clearly to have been accounted for. x x x
the President that the Constitution entrusts the determination of the As for the Mangudadatus, they have an estimated one thousand
need for calling out the armed forces to prevent and suppress eight hundred (1,800) personnel, with about two hundred (200)
lawless violence. Unless it is shown that such determination was firearms. x x x
attended by grave abuse of discretion, the Court will accord Apart from their own personal forces, both clans have Special
respect to the President’s judgment. Civilian Auxiliary Army (SCAA) personnel who support them:
Petitioners contend that the President unlawfully exercised about five hundred (500) for the Ampatuans and three hundred
emergency powers when she ordered the deployment of AFP and (300) for the Mangudadatus.
PNP personnel in the places mentioned in the proclamation.16 But What could be worse than the armed clash of two warring clans
such deployment is not by itself an exercise of emergency powers and their armed supporters, especially in light of intelligence
as understood under Section 23 (2), Article VI of the Constitution, reports on the potential involvement of rebel armed groups
which provides: (RAGs).
SECTION 23. x x x (2) In times of war or other national
emergency, the Congress may, by law, authorize the President, for
a limited period and subject to such restrictions as it may In other words, the imminence of violence and anarchy at the
prescribe, to exercise powers necessary and proper to carry out a time the President issued Proclamation 1946 was too grave to
declared national policy. Unless sooner withdrawn by resolution of ignore and she had to act to prevent further bloodshed and
the Congress, such powers shall cease upon the next adjournment hostilities in the places mentioned. Progress reports also
thereof. indicated that there was movement in these places of both high-
The President did not proclaim a national emergency, only a state powered firearms and armed men sympathetic to the two
of emergency in the three places mentioned. And she did not act clans.23 Thus, to pacify the people’s fears and stabilize the
pursuant to any law enacted by Congress that authorized her to situation, the President had to take preventive action. She called
exercise extraordinary powers. The calling out of the armed forces out the armed forces to control the proliferation of loose firearms
to prevent or suppress lawless violence in such places is a power and dismantle the armed groups that continuously threatened the
that the Constitution directly vests in the President. She did not peace and security in the affected places.
need a congressional authority to exercise the same. 13. Fortun vs Macapagal Arroyo
The President’s call on the armed forces to prevent or suppress Fortun vs. Gloria Macapagal Arroyo
lawless violence springs from the power vested in her under Facts:
Section 18, Article VII of the Constitution, which provides.
• On Novemeber 23, 2009 heavily armed men, believed led by
SECTION 18. The President shall be the Commander-in-Chief of the ruling Ampatuan Family, gunned down and buried under
all armed forces of the Philippines and whenever it becomes shoveled dirt 57 innocent civilians on a highway in
necessary, he may call out such armed forces to prevent or Maguindanao. 
suppress lawless violence, invasion or rebellion. x x x • November 24, 2009, issued Presidential Proclamation 1946,
While it is true that the Court may inquire into the factual bases for declaring a state of emergency in Maguindanao, Sultan Kudarat
the President’s exercise of the above power,18 it would generally and Cotabato City
defer to her judgment on the matter. As the Court acknowledged in • December 4, 2009, President Arroyo issued Presidential
Integrated Bar of the Philippines v. Hon. Zamora,19 it is clearly to Proclamation 1959 declaring martial law and suspending the
the President that the Constitution entrusts the determination of the privilege of the writ of habeas corpus. 
need for calling out the armed forces to prevent and suppress • Two days later or on December 6, 2009, President Arroyo
lawless violence. Unless it is shown that such determination was submitted her report to the congress in accordance with Section
attended by grave abuse of discretion, the Court will accord 18, Art. VII of the 1987 Constitution which required her, within 48
respect to the President’s judgment. hours from proclamation of martial law or the suspension of the
privilege of the writ of habeas corpus, to submit to that body a
The President, as Commander-in-Chief has a vast intelligence report in person or in writing of her action. 
network to gather information, some of which may be classified as • On December 9, 2009 Congress, in joint session, convened
highly confidential or affecting the security of the state. In the pursuant to Section 18, Article VII of 1987 Constitution to review
exercise of the power to call, on-the-spot decisions may be the validity of the Presidents action. 
imperatively necessary in emergency situations to avert great loss • Two days later or on December 12 before Congress could act,
of human lives and mass destruction of property. Indeed, the the President issued Presidential Proclamation 1963, lifting
decision to call out the military to prevent or suppress lawless Martial Law and restoring the privilege of the writ of habeas
corpus in Maguindanao. 

P a g e 12 | 19
POLITICAL LAW REVIEW: IV. Executive Department Case Digests | Wednesday Class @ 4:30-9:30PM

• Petitioners brought the present actions to challenge the Petitioners argue that the general waiver of claims made by the
constitutionality of President Arroyo Proclamation 1959 affecting Philippine government in the Treaty of Peace with Japan is void. They
Maguindanao. But given the prompt lifting of that proclamation claim that the comfort women system established by Japan, and the
before congress could review it and before any serious question brutal rape and enslavement of petitioners constituted a crime against
affecting the rights and liberties of Maguindanao inhabitants could humanity, sexual slavery, and torture. They allege that the prohibition
rise, the court deems any review of its constitutionality the against these international crimes is jus cogens norms from which no
equivalent of beating a dead horse. derogation is possible; as such, in waiving the claims of Filipina comfort
women and failing to espouse their complaints against Japan, the
ISSUE:
Philippine government is in breach of its legal obligation not to afford
Whether or not the validity of the Presidents’s proclamation of impunity for crimes against humanity. Finally, petitioners assert that the
martial law or suspension of the privilege of the writ of habeas Philippine governments acceptance of the apologies made by Japan as
corpus is a political question well as funds from the Asian Womens Fund (AWF) were contrary to
international law.
 
Held: Yes. Respondents maintain that all claims of the Philippines and its nationals
It is evident that under the 1987 Constitution the President and the relative to the war were dealt with in the San Francisco Peace Treaty of
Congress act in tandem in exercising the power to proclaim martial 1951 and the bilateral Reparations Agreement of 1956.  
law or suspend the privilege of the writ if habeas corpus. They  
exercise the power, not only sequentially, but in a sense jointly In addition, respondents argue that the apologies made by Japan have
since, after the President has initiated the proclamation or the been satisfactory, and that Japan had addressed the individual claims
suspension, only the Congress can maintain the same based on its of the women through the atonement money paid by the Asian
own evaluation of the situation on the ground, a power that the Womens Fund.
President does not have.  
The Asian Women's Fund was established by the Japanese
Consequently, although the Constitution reserves to the Supreme government in 1995. The AWF represented the government's concrete
Court the power to review the sufficiency of the factual basis of the attempt to address its moral responsibility by offering monetary
proclamation or suspension in a proper suit, it is implicit that the compensation to victims of the comfort women system. The purpose of
Court must allow Congress to exercise its own review powers, the AWF was to show atonement of the Japanese people through
which is automatic rather than initiated. Only when Congress expressions of apology and remorse to the former wartime comfort
defaults in its express duty to defend the Constitution through such women, to restore their honor, and to demonstrate Japans strong
review should the Supreme Court step in as its final rampant. The respect for women.
constitutional validity of the Presidents proclamation of martial law  
or suspension of the writ of habeas corpus is first a political On January 15, 1997, the AWF and the Philippine government signed a
question in the hands of Congress before it becomes a justiciable Memorandum of Understanding for medical and welfare support
one in the hands of the Court. programs for former comfort women. Over the next five years, these
Here, President Arroyo withdrew Proclamation 1959 before joint were implemented by the Department of Social Welfare and
houses of Congress, which had in fact convened, could act on the Development.
same. Consequently, the petitions in these cases have become   
moot and the Court has nothing to review. The lifting of martial law ISSUE: Whether the Executive Department committed grave abuse of
and restoration of the privilege of the writ of habeas corpus in discretion in not espousing petitioner’s claims for official apology and
Maguindanao was a supervening event that obliterated any other forms of reparations against Japan.
justiciable question.  
HELD: The petition lacks merit.
14. Vinuya vs Romulo  
VINUYA v ROMULO From a
GR No. 162230 April 28, 2010 Domestic
Law
FACTS: Perspectiv
This is an original Petition for Certiorari under Rule 65 of the Rules of e, the
Court with an application for the issuance of a writ of preliminary Executive
mandatory injunction against the Office of the Executive Secretary, the Departme
Secretary of the Department of Foreign Affairs (DFA), the Secretary of the nt has the
Department of Justice (DOJ), and the Office of the Solicitor General exclusive
(OSG). prerogativ
  e to
Petitioners are all members of the MALAYA LOLAS, a non-stock, determine
non-profit organization registered with the SEC, established for the whether to
purpose of providing aid to the victims of rape by Japanese military espouse
forces in the Philippines during the Second World War. petitioners
Petitioners claim that since 1998, they have approached the Executive claims
Department through the DOJ, DFA, and OSG, requesting assistance in against  Ja
filing a claim against the Japanese officials and military officers who pan.
ordered the establishment of the comfort women stations in   
the Philippines. However, officials of the Executive Department declined In Taada v. Cuenco, we held that political questions refer "to those
to assist the petitioners, and took the position that the individual claims of questions which, under the Constitution, are to be decided by the
the comfort women for compensation had already been fully satisfied people in their sovereign capacity, or in regard to which full discretionary
by Japans compliance with the Peace Treaty between authority has been delegated to the legislative or executive branch of
the Philippines and Japan. the government. It is concerned with issues dependent upon
the wisdom, not legality of a particular measure."
P a g e 13 | 19
POLITICAL LAW REVIEW: IV. Executive Department Case Digests | Wednesday Class @ 4:30-9:30PM

  by a coordinate political branch to which authority to make that judgment


Certain types of cases often have been found to present political has been constitutionally committed.
questions. One such category involves questions of foreign relations. It is  
well-established that "[t]he conduct of the foreign relations of our In any event, it cannot reasonably be maintained that the Philippine
government is committed by the Constitution to the executive and government was without authority to negotiate the Treaty of Peace
legislative--'the political'--departments of the government, and the propriety with Japan.
of what may be done in the exercise of this political power is not subject to  
judicial inquiry or decision." The US Supreme Court has further cautioned Indeed, except as an agreement might otherwise provide, international
that decisions relating to foreign policy settlements generally wipe out the underlying private claims, thereby
  terminating any recourse under domestic law. 
are delicate, complex, and involve large elements of  
prophecy. They are and should be undertaken only Respondents explain that the Allied Powers concluded the Peace
by those directly responsible to the people whose Treaty with Japan not necessarily for the complete atonement of the
welfare they advance or imperil. They are decisions of suffering caused by Japanese aggression during the war, not for the
a kind for which the Judiciary has neither aptitude, payment of adequate reparations, but for security purposes. The treaty
facilities nor responsibility. sought to prevent the spread of communism in Japan, which occupied a
  strategic position in the Far East. Thus, the Peace Treaty compromised
To be sure, not all cases implicating foreign relations present political individual claims in the collective interest of the free world.
questions, and courts certainly possess the authority to construe or  
invalidate treaties and executive agreements. However, the question We thus hold that, from a municipal law perspective, that certiorari will
whether the Philippine government should espouse claims of its nationals not lie. As a general principle and particularly here, where such an
against a foreign government is a foreign relations matter, the authority for extraordinary length of time has lapsed between the treatys conclusion
which is demonstrably committed by our Constitution not to the courts but and our consideration the Executive must be given ample discretion to
to the political branches. In this case, the Executive Department has assess the foreign policy considerations of espousing a claim against
already decided that it is to the best interest of the country to waive all Japan, from the standpoint of both the interests of the petitioners and
claims of its nationals for reparations against Japan in the Treaty of Peace those of the Republic, and decide on that basis if apologies are
of 1951. The wisdom of such decision is not for the courts to sufficient, and whether further steps are appropriate or necessary.
question. Neither could petitioners herein assail the said determination by  
the Executive Department via the instant petition for certiorari.
  The Philip
In the seminal case of US v. Curtiss-Wright Export Corp., the US pines is
Supreme Court held that [t]he President is the sole organ of the nation in not under
its external relations, and its sole representative with foreign relations. any
  internation
This ruling has been incorporated in our jurisprudence through Bayan v. al
Executive Secretary and Pimentel v. Executive Secretary; its obligation
overreaching principle was, perhaps, best articulated in (now Chief) to
Justice Punos dissent in Secretary of Justice v. Lantion: espouse
  petitioners
x x x The conduct of foreign relations is full of claims.
complexities and consequences, sometimes with life  
and death significance to the nation especially in times In the international sphere, traditionally, the only means available for
of war. It can only be entrusted to that department of individuals to bring a claim within the international legal system has been
government which can act on the basis of the best when the individual is able to persuade a government to bring a claim on
available information and can decide with the individuals behalf. Even then, it is not the individuals rights that are
decisiveness. x x x It is also the President who being asserted, but rather, the states own rights.
possesses the most comprehensive and the most
confidential information about foreign countries for our The International Law Commissions (ILCs) Draft Articles on Diplomatic
diplomatic and consular officials regularly brief him on Protection fully support this traditional view. They (i) state that "the right
meaningful events all over the world. He has also of diplomatic protection belongs to or vests in the State, (ii) affirm its
unlimited access to ultra-sensitive military intelligence discretionary nature by clarifying that diplomatic protection is a
data.In fine, the presidential role in foreign affairs is "sovereign prerogative" of the State; and (iii) stress that the state "has
dominant and the President is traditionally accorded a the right to exercise diplomatic protection on behalf of a national. It is
wider degree of discretion in the conduct of foreign under no duty or obligation to do so."
affairs. The regularity, nay, validity of his actions are  
adjudged under less stringent standards, lest their Even the invocation of jus cogens norms and erga omnes obligations
judicial repudiation lead to breach of an international will not alter this analysis. Even if we sidestep the question of
obligation, rupture of state relations, forfeiture of whether jus cogens norms existed in 1951, petitioners have not deigned
confidence, national embarrassment and a plethora of to show that the crimes committed by the Japanese army violated jus
other problems with equally undesirable cogens prohibitions at the time the Treaty of Peace was signed, or that
consequences. the duty to prosecute perpetrators of international crimes is an erga
  omnes  obligation or has attained the status of jus cogens.
   
The Executive Department has determined that taking up petitioners The term erga omnes (Latin: in relation to everyone) in international law
cause would be inimical to our country’s foreign policy interests, and could has been used as a legal term describing obligations owed by States
disrupt our relations with Japan, thereby creating serious implications for towards the community of states as a whole. The concept was
stability in this region. For us to overturn the Executive Departments recognized by the ICJ in Barcelona  Traction:
determination would mean an assessment of the foreign policy judgments  

P a g e 14 | 19
POLITICAL LAW REVIEW: IV. Executive Department Case Digests | Wednesday Class @ 4:30-9:30PM

x x x an essential distinction should be drawn Yes. The President may enter into an executive agreement on
between the obligations of a State towards the foreign military bases, troops, or facilities.
international community as a whole, and those arising
vis--vis another State in the field of diplomatic 1. The role of the President as
protection. By their very nature, the former are the the executor of the law
concern of all States. In view of the importance of the includes the duty to defend
rights involved, all States can be held to have a legal the State, for which purpose
interest in their protection; they are obligations erga he may use that power in the
omnes. conduct of foreign relations
  SECTION 1. Power of Control. —
Such obligations derive, for example, in contemporary The President shall have control of all the
international law, from the outlawing of acts of executive departments, bureaus, and
aggression, and of genocide, as also from the offices. He shall ensure that the laws be
principles and rules concerning the basic rights of the faithfully executed. 
human person, including protection from slavery and
racial discrimination. Some of the corresponding rights Hence, the duty to faithfully execute the laws of the
of protection have entered into the body of general land is inherent in executive power and is intimately related
international law others are conferred by international to the other executive functions. These functions include the
instruments of a universal or quasi-universal faithful execution of the law in autonomous regions; the right
character. to prosecute crimes;  the implementation of transportation
   projects; the duty to ensure compliance with treaties,
The Latin phrase, erga omnes, has since become one of the rallying cries executive agreements and executive orders;  the authority to
of those sharing a belief in the emergence of a value-based international deport undesirable aliens;  the conferment of national
public order. However, as is so often the case, the reality is neither so awards under the President's jurisdiction;  and the overall
clear nor so bright. Whatever the relevance of obligations erga omnes as administration and control of the executive department. 
a legal concept, its full potential remains to be realized in practice.
The import of this characteristic is that the manner
of the President's execution of the law, even if not expressly
The term is closely connected with the international law concept of jus
granted by the law, is justified by necessity and limited only
cogens. In international law, the term jus cogens (literally, compelling law)
by law, since the President must "take necessary and proper
refers to norms that command peremptory authority, superseding
steps to carry into execution the law
conflicting treaties and custom. Jus cogens norms are considered
peremptory in the sense that they are mandatory, do not admit In light of this constitutional duty, it is the
derogation, and can be modified only by general international norms of President's prerogative to do whatever is legal and
equivalent authority. necessary for Philippine defense interests. It is no
  coincidence that the constitutional provision on the faithful
 Of course, we greatly sympathize with the cause of petitioners, and execution clause was followed by that on the President's
we cannot begin to comprehend the unimaginable horror they commander-in-chief powers, which are specifically granted
underwent at the hands of the Japanese soldiers. We are also during extraordinary events of lawless violence, invasion, or
deeply concerned that, in apparent contravention of fundamental rebellion. And this duty of defending the country is
principles of law, the petitioners appear to be without a remedy to unceasing, even in times when there is no state of lawless
challenge those that have offended them before appropriate violence, invasion, or rebellion. At such times, the President
fora. Needless to say, our government should take the lead in has full powers to ensure the faithful execution of the laws. 
protecting its citizens against violation of their fundamental human
rights. Regrettably, it is not within our power to order the Executive It would therefore be remiss for the President and
Department to take up the petitioners cause. Ours is only the power repugnant to the faithful-execution clause of the Constitution
to urge  and exhort the Executive Department to take up petitioners to do nothing when the call of the moment requires
cause. increasing the military's defensive capabilities, which could
include forging alliances with states that hold a common
15. Saguisag vs Ochoa interest with the Philippines or bringing an international suit
SAGUISAG vs. OCHOA against an offending state.
[G.R. No. 212444. January 12, 2016.] 2. The plain meaning of the
Constitution prohibits the entry
Facts: of foreign military bases, troops
or facilities, except by way of a
The petitions question the constitutionality of the
treaty concurred in by the
Enhanced Defense Cooperation Agreement (EDCA) between the
Senate — a clear limitation on
Republic of the Philippines and the United States of America
the President's dual role as
(U.S.). Petitioners allege that respondents committed grave abuse
defender of the State and as sole
of discretion amounting to lack or excess of jurisdiction when they
authority in foreign relations.
entered into EDCA with the U.S., claiming that the instrument
violated multiple constitutional provisions. To support the legality of The initial limitation is found in Section 21 of the
their actions, respondents invoke the 1987 Constitution, treaties, provisions on the Executive Department: "No treaty or
and judicial precedents. international agreement shall be valid and effective unless
concurred in by at least two-thirds of all the Members of the
Issues: Senate." The specific limitation is given by Section 25 of the
Transitory Provisions, the full text of which reads as follows:
Whether the President may enter into an executive agreement on
foreign military bases, troops, or facilities SECTION 25. After the expiration
in 1991 of the Agreement between the
Ruling:
P a g e 15 | 19
POLITICAL LAW REVIEW: IV. Executive Department Case Digests | Wednesday Class @ 4:30-9:30PM

Republic of the Philippines and the United the Constitution and Philippine law, and not to the Section
States of America concerning Military 25 requirement of validity through a treaty.
Bases, foreign military bases, troops, or
facilities shall not be allowed in the Moreover, the Court indicated that the Constitution
Philippines except under a treaty duly continues to govern the conduct of foreign military troops in
concurred in by the Senate and, when the the Philippines, readily implying the legality of their initial
Congress so requires, ratified by a majority entry into the country.
of the votes cast by the people in a national As applied, verba legis aids in construing the
referendum held for that purpose, and ordinary meaning of terms. In this case, the phrase being
recognized as a treaty by the other construed is "shall not be allowed in the Philippines" and not
contracting State. the preceding one referring to "the expiration in 1991 of the
It is quite plain that the Transitory Provisions of the Agreement between the Republic of the Philippines and the
1987 Constitution intended to add to the basic requirements of United States of America concerning Military Bases, foreign
a treaty under Section 21 of Article VII. This means that both military bases, troops, or facilities." It is explicit in the
provisions must be read as additional limitations to the wording of the provision itself that any interpretation goes
President's overarching executive function in matters of beyond the text itself and into the discussion of the framers,
defense and foreign relations.  the context of the Constitutional Commission's time of
drafting, and the history of the 1947 MBA. Without reference
3. The President, however, may to these factors, a reader would not understand those terms.
enter into an executive However, for the phrase "shall not be allowed in the
agreement on foreign military Philippines," there is no need for such reference. The law is
bases, troops, or facilities, if (a) clear. No less than the Senate understood this when it
it is not the instrument that ratified the VFA.
allows the presence of foreign
military bases, troops, or 4. The President may generally
facilities; or (b) it merely aims enter into executive
to implement an existing law or agreements subject to
treaty. limitations defined by the
Constitution and may be in
SECTION 25. After the expiration furtherance of a treaty
in 1991 of the Agreement between the already concurred in by the
Republic of the Philippines and the United Senate.
States of America concerning Military
Bases, foreign military bases, troops, or As the sole organ of our foreign relations and the
facilities shall not be allowed in the constitutionally assigned chief architect of our foreign
Philippines except under a policy,  the President is vested with the exclusive power to
treaty duly concurred in by the Senate and, conduct and manage the country's interface with other states
when the Congress so requires, ratified by a and governments. Being the principal representative of the
majority of the votes cast by the people in a Philippines, the Chief Executive speaks and listens for the
national referendum held for that purpose, nation; initiates, maintains, and develops diplomatic relations
and recognized as a treaty by the other with other states and governments; negotiates and enters
contracting State. (Emphases supplied) into international agreements; promotes trade, investments,
tourism and other economic relations; and settles
A plain textual reading of Article XIII, Section 25, international disputes with other states.
inevitably leads to the conclusion that it applies only to a
proposed agreement between our government and a foreign The President has the prerogative to
government, whereby military bases, troops, or facilities of conclude binding executive agreements that do not require
such foreign government would be "allowed" or would "gain further Senate concurrence. The existence of this
entry" Philippine territory. presidential power is so well-entrenched that Section 5 (2)
(a), Article VIII of the Constitution, even provides for a check
Note that the provision "shall not be allowed" is a on its exercise. As expressed below, executive agreements
negative injunction. This wording signifies that the President is are among those official governmental acts that can be the
not authorized by law to allow foreign military bases, troops, or subject of this Court's power of judicial review:
facilities to enter the Philippines, except under a treaty
concurred in by the Senate. Hence, the constitutionally (2) Review, revise, reverse, modify, or
restricted authority pertains to the entry of the bases, troops, affirm on appeal or certiorari, as the
or facilities, and not to the activities to be done after entry. law or the Rules of Court may
provide, final judgments and orders
Under the principles of constitutional construction, of of lower courtsin:
paramount consideration is the plain meaning of the language
(a) All cases in which
expressed in the Constitution, or the verba legis rule. It is
the constitutionality or
presumed that the provisions have been carefully crafted in
validity of any treaty, intern
order to express the objective it seeks to attain.  It is
ational or executive
incumbent upon the Court to refrain from going beyond the
agreement, law,
plain meaning of the words used in the Constitution.
presidential decree,
It is evident that the constitutional restriction refers proclamation, order,
solely to the initial entry of the foreign military bases, troops, or instruction, ordinance, or
facilities. Once entry is authorized, the subsequent acts are regulation is in question.
thereafter subject only to the limitations provided by the rest of (Emphases supplied)

P a g e 16 | 19
POLITICAL LAW REVIEW: IV. Executive Department Case Digests | Wednesday Class @ 4:30-9:30PM

One of the distinguishing features of executive 1. Section 25, Article XVIII of the Constitution,
agreements is that their validity and effectivity are not affected contains stringent requirements that must
by a lack of Senate concurrence. be fulfilled by the international agreement
allowing the presence of foreign military
Executive agreements may dispense with the bases, troops, or facilities in the
requirement of Senate concurrence because of the legal Philippines: (a) the agreement must be in
mandate with which they are concluded. the form of a treaty, and (b) it must be duly
There are constitutional provisions that restrict or limit concurred in by the Senate.
the President's prerogative in concluding international 2. If the agreement is not covered by the above
agreements, such as those that involve the following: situation, then the President may choose
a. The policy of freedom from nuclear weapons within the form of the agreement (i.e., either an
Philippine territory  executive agreement or a treaty), provided
that the agreement dealing with foreign
b. The fixing of tariff rates, import and export quotas, military bases, troops, or facilities is not
tonnage and wharfage dues, and other the principal agreement that first allows
duties or imposts, which must be pursuant their entry or presence in the Philippines. 
to the authority granted by Congress 
3. The executive agreement must not go beyond
c. The grant of any tax exemption, which must be the parameters, limitations, and standards
pursuant to a law concurred in by a majority set by the law and/or treaty that the former
of all the Members of Congress purports to implement; and must not
d. The contracting or guaranteeing, on behalf of the unduly expand the international obligation
Philippines, of foreign loans that must be expressly mentioned or necessarily
previously concurred in by the Monetary implied in the law or treaty.
Board  4. The executive agreement must be consistent
e. The authorization of the presence of foreign with the Constitution, as well as with
military bases, troops, or facilities in the existing laws and treaties.
country must be in the form of a treaty duly In light of the President's choice to enter into EDCA
concurred in by the Senate.  in the form of an executive agreement, respondents carry
f. For agreements that do not fall under paragraph 5, the burden of proving that it is a mere implementation of
the concurrence of the Senate is required, existing laws and treaties concurred in by the Senate. EDCA
should the form of the government chosen must thus be carefully dissected to ascertain if it remains
be a treaty. within the legal parameters of a valid executive agreement.

5. The President had the choice


to enter into EDCA by way of
an executive agreement or a
treaty.
No court can tell the President to desist from
choosing an executive agreement over a treaty to embody an
international agreement, unless the case falls squarely within
Article VIII, Section 25.
Indeed, in the field of external affairs, the President
must be given a larger measure of authority and wider
discretion, subject only to the least amount of checks and
restrictions under the Constitution.
Section 9 of Executive Order No. 459, or the
Guidelines in the Negotiation of International Agreements and
its Ratification, thus, correctly reflected the inherent powers of
the President when it stated that the DFA "shall determine
whether an agreement is an executive agreement or a treaty."
Accordingly, in the exercise of its power of judicial
review, the Court does not look into whether an international
agreement should be in the form of a treaty or an executive
agreement, save in cases in which the Constitution or a
statute requires otherwise. Rather, in view of the vast
constitutional powers and prerogatives granted to the
President in the field of foreign affairs, the task of the Court is
to determine whether the international agreement is consistent
with the applicable limitations.
6. Executive agreements may
cover the matter of foreign
military forces if it merely
involves detail adjustments. 

P a g e 17 | 19
POLITICAL LAW REVIEW: IV. Executive Department Case Digests | Wednesday Class @ 4:30-9:30PM

P a g e 18 | 19
POLITICAL LAW REVIEW: IV. Executive Department Case Digests | Wednesday Class @ 4:30-9:30PM

P a g e 19 | 19

You might also like